« first day (3031 days earlier)      last day (1991 days later) » 

3:06 PM
To put it another way: Suppose you have three nonzero vectors $a,b,c$ such that $a$ is collinear to both $b$ and $c$. Then $b,c$ are also collinear.
 
Does anyone know how to write the proof for "Is a graph with 8 vertices and 6 edges connected". I would think it's a no, because my intuition is that the longest connected graph I think is a linear tree, and such a graph with n vertices has n-1 edges ... so I would expect 7 vertices and 6 edges ... then 1 vertex will be left out
 
@PrashinJeevaganth another approach might be: What's the smallest number of edges needed to create a connected graph on eight vertices?
Presumably the point is that six edges is not enough (whereas seven edges is enough since then you indeed can construct a linear tree)
 
OK, I've read this until the bit on quantum mechanics
and I'm in a superposition of understanding and not understanding
 
oh, neat, pictorial QM
 
Like, I get what the diagrams are doing
I don't get what the physical interpretations are
 
3:15 PM
@Semiclassical I would think that will be 7, but how does arguing about not having enough vertices change a thing from having not enough edges?
 
Well, suppose you can prove that you need at least 7 edges in order to have a connected graph on 8 vertices.
Then a graph with only 6 edges on 8 vertices can't be connected.
 
Prove that, when you delete an edge, the number of components of the graph either stays the same or increases by 1
 
oh, good call
 
and note that, when you delete all 6 edges, you have 8 components
 
@AkivaWeinberger part of what's going on is how measurement works in QM
 
3:17 PM
so you must have started with at least 2 components
I guess there's the reverse argument as well
 
Suppose you start with a state $|\psi\rangle$, then make a measurement of an observable $A$ and obtain the state $|\phi\rangle.$
 
When you add an edge, the number of components either stays the same or decreases by one
and then you start from the 8 disjoint points and add the 6 edges one-by-one
 
The measurement of $A$ therefore projected your initial state onto the final state.
 
and you can't decrease the connected components from 8 to 1 since it can at most go down to 2
Same argument, really
@Semiclassical OK
 
And the simplest way to write that down is that $|\psi\rangle \mapsto |\phi\rangle \langle \phi|\psi\rangle$
 
3:21 PM
Is $\langle\phi|\psi\rangle$ a scalar?
 
Yeah
$|\psi\rangle$ is a vector, whereas $\langle \psi|$ is its dual
 
Apparently if you take all the numbers from $1$ to $10^{10}$, write them out as words, and order them alphabetically, the first odd number is just over $8$ million
No, $8$ billion
 
Isn't "eight" before "eight billion" alphabetically
 
So in terms of vectors $u,v$ we've got $u\mapsto (v v^*) u=(v^* u)v$
 
(Also, I think "octillion" is the smallest number with a 'c' in it)
 
3:23 PM
which indeed lies in the subspace generated by $v$ so long as $v^*u\neq 0$
 
Ah, so $|\phi\rangle\langle\phi|$ is like a matrix?
 
@Akiva didn't know that
lo
little factz
 
actually, I guess it's true even if that's zero, but only in the trivial sense
@AkivaWeinberger yep
 
Hi,

I'm here to ask if anyone has seen a function like this before:

$f(x) = \sum_{i=1}^{\infty} x^{q^i}$
 
@Breakfastisready oh hey, lacunary series
 
3:24 PM
Oh. Thanks, that's what I needed. A starting point.
 
What does the scalar $\langle\phi|\psi\rangle$ represent? Is it a probability of $\phi$ happening?
@Breakfastisready Also I think the Jacobi theta functions are related but I don't really remember what they are
 
Not quite. Lemme state it more clearly.
Suppose you start with an initial state $|\psi\rangle$ and perform a measurement $A$, yielding outcomes $\{\lambda_i\}_{i=1}^n$
 
@Semiclassical I'm listening
 
lol, you're both waiting on me
I'll assume that it's a projective measurement, i.e. once you've measured $A$ and gotten $\lambda_i$ as the outcome, it should be the case that measuring $A$ again should still give the same outcome.
(On the other hand, if you measured A and then made a different measurement B, then A again, you wouldn't necessarily get the same result for A twice.)
 
@Breakfastisready He was talking to me
 
3:28 PM
What that imposes is that measuring $A$ and receiving $\lambda_i$ places us in some state with a definite value of $A$
 
but I think your function is a type of lacunary function, but lacunary functions are more general
 
which basically requires that the new state be an eigenstate of the Hermitian operator corresponding to observable $A$
 
i.e. $A|\phi_i\rangle = \lambda_i |\phi_i\rangle$
 
@AkivaWeinberger I see, thanks.
 
3:30 PM
Now, typically such eigenstates form a complete basis
 
So the results of our observations are scalars?
 
typically, yeah
you can have vector results under some conditions, but that requires further conditions
 
So $A$ takes a state as the input, and gives us the product of the state and our observation of it as output
or something
 
basically, you need to be able to measure each component of your vector individually without interfering with the other components
if you can do that, then you can regard your observations as forming a vector. But that's often not true
So usually we just focus on scalars.
anyways. completeness means you can write your initial state as $|\psi\rangle = \sum_{i=1}^n c_i |\phi_i\rangle$
and similarly $\langle \psi|=\sum_{i=1}^n \langle \phi_i|c_i^*$
So $\langle \psi|\psi\rangle =\sum_{i,j=1}^n \langle \phi_i |\phi_j\rangle c_i^* c_j$
But, you furthermore choose your $|\phi_i\rangle$ to give an orthonormal basis
(worth stressing here that observables in QM are assumed to correspond to Hermitian operators. that's what ensures the nice properties of our basis.)
 
Wait, so I have the state $\psi$, and I'm asking it what its $\phi_i$ component is, and the result is $|\phi_i\rangle\langle\phi_i|\psi\rangle$?
 
3:37 PM
Yep.
 
I guess that's like how the $\hat\imath$ component of $\vec v$ is $\hat\imath(\hat\imath\cdot\vec v)$.
 
It's exactly so, since $\langle \phi_i |\phi_j\rangle$ is just the inner product between $\phi_i$ and $\phi_j$
 
So this seems kinda classical then
 
If classical = linear algebra, then yeah
A lot of QM is really just linear algebra
 
But then where does the quantum weirdness like randomness and entanglement come from
 
3:39 PM
anyways. If we assume $\langle \phi_i|\phi_j\rangle = 1$ if $i=j$ and 0 otherwise, then the above collapses to $\langle \psi |\psi\rangle = \sum_{i=1}^n |c_i|^2$
 
or quantization or particle-wave duality or
@Semiclassical Sure
That's the Pythagorean theorem
 
yep
the way one assigns probabilities is that as $p_i = |c_i|^2/\sum_{i=1}^n |c_i|^2$
in which case $\sum_i p_i = 1$ as it should
 
To make the link with what you wrote earlier, suppose we instead computed $\langle \phi_i | \psi\rangle=\sum_{j=1}^n c_j \langle \phi_i |\phi_j\rangle = c_i$
so our probabilities can be conveniently written as $p_i = \frac{|\langle \phi_i |\psi\rangle|^2}{\langle \psi|\psi \rangle}$
To make life easier, one usually goes a step further and scales $|\psi\rangle $ to have unit norm i.e. $\langle \psi|\psi\rangle = 1$
 
It's kind of like if measuring the vector $(3,4)$ had a $9/25$ chance of turning it into $\hat\imath$ and a $16/25$ chance of turning it into $\hat\jmath$
 
3:45 PM
So $p_i = \text{Pr}(A=\lambda_i)=|\langle \phi_i |\psi\rangle|^2$
@AkivaWeinberger pretty much, yeah
 
but only if the measurement $A$ is parallel to $\hat\imath$ or something?
Is this like with polarized light
 
The statement would be that $\hat{i}$ and $\hat{j}$ would be eigenvectors of $A$
 
where I'm passing it through a horizontal filter so it kinda has to choose whether or not it's $\hat\imath$ or $\hat\jmath$
@Semiclassical Ah
 
@AkivaWeinberger pretty much, yeah
And that's a good analogy, in fact.
You can think of unpolarized light as a 1-to-1 mixture of horizontal and vertical polarization
If you put a horizontal polarizer in front of the light, then you're only allowing those photons with horizontal polarization to pass through.
 
but if I rotated my filter by $\arctan(4/3)$, it would be $(3,4)$ with a 100% chance and $(-4,3)$ with a 0% chance
 
3:50 PM
Right.
 
So what's the deal on page 21 where it talks about quantum teleportation in my link
 
@AkivaWeinberger And if you placed a horizontal filter after this one, only 9/25 of the light passing through the first filter would pass through the second
(9/25 in the sense of intensity)
 
Hi chat
 
which is what we would expect if light weren't quantized
The weird thing is that it happens on a photon-by-photon basis
 
i don't have a great sense of quantum teleportation tbh
@AkivaWeinberger yeah
the action of a polarizer is derived from electromagnetic wave theory
so the idea that you can make it work photon-by-photon is bizarre from that point of view
the main nice thing about the polarizer analogy imo is that you can have the following
Suppose you set up a horizontal polarizer and a vertical polarizer in sequence. Then no light should pass through both of them
So you measure polarization, selecting for horizontal polarization, and then measure again, selecting for vertical polarization. You end up with no light going through
 
3:59 PM
Have you seen this YouTube video?
 
Suppose, though, you now insert a third polarizer at some angle $\theta$ between them
No
i'm guessing it's what I'm sayihng?
 
I think so, yeah
They have filters at 0, 45, and 90 degrees in that image
 
I do object to calling that Bell's theorem, though
it's a good analogue of QM, but it really has nothing to do with Bell
 
and with waves, you'd expect more to get through 0-45-90 then just 0-90 (i.e. zero)
but with particles, you wouldn't, or something
or you wouldn't expect to get as much
 
To get Bell you have to start making observations on separate correlated systetms
@akiva to put it mathematically: Suppose you start with a unit vector $\hat{i}$. If you project that onto the vector $\hat{j}$, you get zero
 
4:03 PM
So you project it to $s:=\frac1{\sqrt2}(\hat\imath+\hat\jmath)$ in the middle
 
Right
 
and you get $\frac1{\sqrt2}s$
 
Which in terms of linear algebra is just the fact that projectors are operators and therefore needn't commute
 
and then you project to $\hat\jmath$ and get $\frac12\hat\jmath$
 
Yeah. Note then that $\|\hat{j}/2\|^2=1/4$
whereas initially you had $\|\hat{i}\|^2=1$
You work with the squares so that you add things directly to get the total
 
4:06 PM
Makes sense
 
The problem with this analogy, I should note, is that in QM your measurements aren't of the form "Measure A, and keep it if it comes out as $\lambda$"
they're just "Measure A"
You can of course set up your device as to only keep track of the states which come out as $\lambda$, but that's a further step
 
What is the simplified version of "not all students don't like mathematics"?
 
There exists a student who likes mathematics @ArtificialStupidity
 
For instance, suppose you send a beam of electrons through a Stern-Gerlach device. then you'll end up with two beams of electrons, deflected along two different directions
 
My attempt: Some students don't like mathematics. Is it wrong?s
 
4:10 PM
no
 
Maybe consider the statement "Not all students like art" first.
 
this is all prohibited subject matter I demand the SF ban both just as I was for my awesome response to will kunting being a banana anding also loving said fruit
hunting*
 
What exactly is your problem?
I saw your message. There was nothing awesome about it.
 
I can't actually parse that sentence
 
I just tabbed in and saw this comment and stared at it blankly for over 15 seconds
 
4:13 PM
In other news, Semi is teaching me QM
 
Believe me, it's not better if you actually understand the context.
 
Thank you all!
 
@AkivaWeinberger on a general note, the Stern-Gerlach device is my go-to example for QM systems
 
Hi Balarka
 
Looking it up
 
4:14 PM
Hi Astyx
How's it going
 
Quite good and you ?
It's been quite a long time
 
It's the one where you send electrons through and they either come out deflected up or deflected down
 
It's alright. Semester is over and I'm going home for winter break
 
One thing to note, though: In the typical presentation of the S-G device, one doesn't worry about the electron as a particle moving in space.
 
Nice ! Where are you studying again ?
 
4:16 PM
One is only concerned with "You start with an electron without definite spin-z component and end up with an electron with definite spin-z."
 
So you won't typically see the Schrodinger equation invoked, at least not in the sense of $-\partial_x^2 \psi +V \psi = E\psi$
(You can invoke the Schrodinger equation in that sense, but you'll need to take $\psi$ to be itself a 2-vector and $V$ some matrix. most people don't bother going down that route and just worry about the spin degree of freedom rather than the space degrees)
 
Wait, is spin a rotation matrix?
 
I guess that's a big thing to appreciate about QM: So much of what's going on is to do with what information you treat as relevant and what information you discard as irrelevant
 
or a bivector or something
 
4:20 PM
Depends what you mean by that.
 
Like, classically, angular momentum is a (bi)vector
Is spin an angular momentum dealie
 
Suppose I've got an electron, regarded as a spin-1/2 particle. Then its wavefunction is properly not $\psi(x)$ but $\Psi(x)=\binom{\phi(x)}{\chi(x)}$
In writing that, though, I'm assuming some basis for my spin operators
Typically, one assumes that the operator for the z-component is $S_z = \frac{\hbar}{2}\begin{pmatrix} 1 & 0 \\ 0 & -1\end{pmatrix}$
 
Operator?
 
In the same way as $A$ was the Hermitian operator corresponding to a particular observable
we tend to be rather careless there, though
I.e. we'll refer to A as both the operator and the actual observable
properly it should probably be $\hat{A}$ as the operator and $A$ as the observable
 
"Observable" meaning "observable property"?
 
4:25 PM
More or less, yeah
 
Like spin or charge or position
well, maybe not position
 
no, position is an observable
 
Position and momentum are both observables?
 
there's some subtlety attached to it, since the outcomes are continuous rather than discrete. but it is an observable.
yeah
You get into the weirdness of position measurements when you think about how an actual position detector would work, i.e. a realistic detector is a large but finite set of pixels with some local sensitivity
 
So what does $S_z$ represent
Something about $\pm\hbar/2$
 
4:29 PM
$\hat{S}_z$ is the operator corresponding to the observable $S_z$. you measure it by sending an electron through a stern-gerlach device (oriented in the $\hat{z}$ direction) and observing the deflection direction (up or down); we interpret it as the z-component of the electron's spin angular momentum
 
And it's either 100% spinning clockwise in the xy-plane at some speed or 100% spinning counterclockwise in the xy-plane at that speed
 
That would be the classical model, yes
 
and that speed is $\hbar/2$
 
Not quite. $\hbar$ has units of action (angular momentum) not speed
 
Isn't angular momentum in radians per second
 
4:31 PM
no.
angular momentum = I omega = kg*meters^2/second
 
Oh, it's that times weight times radius
so kg m^2 / s
 
yep
or linear momentum * position
 
So to get the angular velocity I'd need to know the angular momentum of a sphere
 
( angular momentum * angle has the same units as linear momentum * position)
Yeah. And then you'd have to relate that to the magnetic moment of an electron
So the linkage is: A spinning sphere of charge acts like a miniature current loop i.e. a magnetic dipole
And a magnetic dipole, passing through a magnetic field gradient, experiences a force depending on its orientation
the electron acts like it is spinning on its axis with angular momentum $\hbar/2$ and therefore has a certain magnetic moment, which means it'll get deflected by the magnetic field gradient
 
and make two spots on the detector
 
4:35 PM
(that's what the stern-gerlach device is at its heart: a configuration of magnets which produce a magnetic field gradient)
@AkivaWeinberger that's whats seen, yes
The weirdness comes when you probe that interpretation more and try to insist that the electron really is spinning on its axis
you get results which seem absurd
e.g. the electron would have to be spinning at roughly the speed of light iirc
 
Oh wow
What happens if you do it twice
 
Do what twice?
 
Pass the positive spin ones through another Stern–Gerlach machine
 
Ah. If the second device has the same orientation as the first, then it should exit with the same deflection
 
and if it's oriented orthogonally it'll be random
 
4:39 PM
yep
 
with probability $\cos^2(\theta)$ or something
 
right
one fun thing is that the labels for spin-up/down depend so much on orientation
by that, I mean the following
 
Wait, $\cos^2(\theta)$ doesn't make sense since it shouldn't go below $1/2$
Or wait hold on
Oh OK I got it
 
for that detector orientation, you'd get probabilities $\cos^2(\theta)$ for spin-up, $\sin^2(\theta)$ for spin-down (assuming you're sending in electrons which came out spin-up from an SG device oriented in the +z sense)
 
If you turn it 180 degrees, it'll be 100% the "opposite" way
 
4:42 PM
Right.
 
OK I got it now
 
to be more pricse: Suppose you had two electron beams and you fed them into two Stern-Gerlach devices. One is oriented in the positive $z$ direction, whereas the other is flipped upside down.
In both cases, you'll get electrons deflected in the same way.
However, if you send the spin-up beam from the first one into the second one, you'll now find that they're all deflected down
 
Right
It's just a labeling thing
My right is your left
 
Sure. But note that that's quite a difference with a polarizer, where rotating it 180 degrees gives you back what you started with
 
Right
'cause one's checking for unoriented direction and the other's checking for oriented direction
$\rm\Bbb RP^2$ versus $S^2$
 
4:46 PM
yup
in physics, we usually use the word 'director' for the former
 
And for the latter?
 
(direction) vector
 
the fun thing in my book is that the spin-up beam of the +z device is an eigenstate of both the +z device and the -z device
but it doesn't have the same eigenvalue
There's nothing terribly deep about that, of course.
 
$\pm1$?
 
4:48 PM
$\pm \hbar/2$, to be more precise
but frankly I don't worry about that \hbar/2 in practice much
the big exception being when you want to consider particles of arbitrary spin, lol
 
What if the electron emitter is spinning
 
then how do you make sure the electron reaches the device, lol
 
Spinning around the axis of the beam
 
that's just confusing
 
Spinning in the xy-plane
 
4:51 PM
 
I was wondering if it would impart any angular momentum onto the electron
 
typically not, no
 
If you fire a bullet out of a spinning gun you get a spinning bullet
 
Really?
 
(unless the bullet was gonna spin anyway and the gun was spinning in the opposite direction)
 
4:52 PM
Great observation
 
though I'm sure you could find examples where an electron, emitted from a molecule, comes out with some definite spin angular momentum
I don't want to say you can't find examples like that, but it's typically a bit subtler than that
I don't deal with this stuff so much tho
 
I mean, you said it was a ridiculous angular velocity anyway so it probably wouldn't be measurable
 
You could probably do an experiment with observations from pulsars or something
 
you can definitely find experiments involving spin-polarized electrons, to be sure
but I don't think you can do so without involving electromagnetism in some way
mechanical rotation by itself is probably not enough
 
4:57 PM
So so far none of this seems particularly uncomputable
I image that would change once wavefunctions get thrown in
and virtual particles or whatever those are
 
the place things become weird in my book is when you start considering correlated systems
e.g. you entangle two electrons in a singlet state, then move them apart and separately measure them
 
Could you measure one in the z-axis and the other in the y-axis or something
Oh, wait, that doesn't do what I thought it would
Entangled polarized light would be better
Polarized at a 45deg angle
 
typically you'd give the observers three stern-gerlach devices each
 
If you measure one through a vertical filter than it should become purely vertical
 
one oriented in the +z direction and the other two oriented at 120 degree angles in the plane
 
5:01 PM
and then if you measure the other through a horizontal filter, because of the first one it should not go through the filter 100% of the time
 
(to get a sense of why that's an interesting case, note that the vectors for those three directions add to zero. up to rotation, that's actually the only way to add three unit vectors together and get zero)
 
Right, this I know
 
the place where 45 degree angles emerge is more if you were dealing with the so-called CHSH inequality
 
@Semiclassical That's the same as saying the only triangle with three unit sides is the equilateral triangle
 
yup
in the CHSH setup, you'd have one observer who can measure Sx, Sy, while the other observer can measure along Sx',Sy'. specifically, you'd end up picking x'=(x+y)/sqrt(2) and y'=(-x+y)/sqrt(2)
 
5:04 PM
Wait hold on
Can we go back to the polarized light setup for a second
So the light starts 45deg polarized and it's entangled
One goes through a vertical filter, two possibilities
 
No.
In order to have entanglement, you need to have two systems
In the polarized light setup, you just have the one.
 
You can't polarize two photons?
 
Oh
Sure, you can do that
 
*entangle
 
then you'd send the two photons through separate polarizers
 
5:06 PM
I meant entangle, sorry
 
and they'd start out as entangled
yeah, I understood you
 
Oh OK
So one goes through a vertical filter, say it goes through
Does the other one have to get blocked?
If the other one goes through a horizontal filter
And if the first one gets blocked by the vertical filter than does the second one go through the horizontal filter
 
it depends on how you've entangled them
But that's certainly one way they could be entangled
 
I realize this is actually useless at sending information
since you don't know whether mine has been blocked or not
 
Right
 
5:08 PM
so let's go back to the Stern–Gerlach thing
 
sure
In that case, if both of us measure along the +z direction, and our electrons were entangled in the singlet state
then one of us will get positive deflection and the other will get negative deflection
 
Oh, so they're entangled oppositely
Fine
 
yeah. a singlet state has zero total angular momentum
 
Now let's measure 120degrees down-right
 
sure. if I measure positive deflection, then the probability of you measuring negative deflection will be 1/4 iirc
To make this more quantitative, let your observables be $S_{a1},S_{a2},S_{a3}$ and mine be $S_{b1},S_{b2},S_{b3}$
If we measure along the same direction, we definitely get opposite results. For convenience, let me label these outcomes as $\pm 1$ rather than $\pm \hbar/2$
 
5:18 PM
I could rotate my SG machines 180 degrees preemptively
 
so, some relevant statistics: you'll always have $S_{a1}^2=1$, so $\langle S_{a1}^2\rangle =1$ and the same for the rest of our observables
Yeah. Equivalently, I could take my results and guess that your result is the opposite of mine
Which will work perfectly when we happen to pick the same direction
For now, though, let's leave them as written
In which case we always have $S_{a1}S_{b1}=-1\implies \langle S_{a1}S_{b1}\rangle =-1$
$\langle A\rangle$ is the usual physics way of writing the expected value of the observable $A$. that arises from the fact that $\langle A\rangle =\frac{\langle \psi | A|\psi\rangle}{\langle \psi|\psi\rangle}$, so it's handy notation
anyways. my statement above implies that $\langle S_{a1}S_{b2}\rangle = \frac14 (-1)+\frac34 (+1)=\frac12$
and that remains true if I replace 1,2 with another other pair of distinct indices
 
Unrelated: under the set-theoretic definition of a function, $f\cap g$ is a function whose domain is the set on which $f$ and $g$ agree
 
5:35 PM
Let $N$ be the normal distribution. If $a \sim N(0,A)$ and $b \mid a \sim N(a,B)$ then $b \sim N(0,A+B)$ right?
 
What does b|a represent?
I'm not familiar with that notation
 
$b$ given $a$. So it's a normal distribution whose mean is itself normally distributed.
 
And I'm trying to marginalize over $a$, i.e. $p(b) = \int p(b \mid a) p(a) \,\mathrm{d}a$
 
that E[b]=0 is intuitive enough
Your problem should be equivalent to some integral identity, shouldn't it?
 
5:39 PM
I used the following reasoning: First, $b = b - a + a$. Since $b-a$ and $a$ are normally distributed, $b$ must be normally distributed.
Second, $E[b] = E[b-a+a] = E[b-a]+E[a] = 0+0 =0$ by the linearity of expectation.
 
that's probably slicker than what I'm suggesting
 
Third, since $b-a$ and $a$ are independent (?) then $Var[b] = Var[b-a+a] = Var[b-a] + Var[a] = B + A$.
 
Geometrically it seems to make sense, you're "expanding" the original covariance.
 
@Semiclassical Wait were you gonna finish this
 
5:42 PM
sorta lost track of it, yeah
@user76284 one thing that's a bit confusing about this: is $a$ the r.v. itself, or a particular value of it
 
The random variable
 
then you really shouldn't be integrating with respect to it
 
It's a common abuse of notation for $p(b = x) = \int p(b = x \mid a = y) p(a = y) \,\mathrm{d}y$
 
if you've got $X$ as a continuous random variable, you don't write the expectation of $f$ as $\int f(X)\,dX$
yeah, fair
 
I need to go now actually
 
5:44 PM
@AkivaWeinberger ok. bug me about it later
@user76284 I'm more used to writing r.v.s as capital letters and their values as lower case, e.g. $p(a)=\text{Pr}(A=a)$ for a discrete r.v. $A$
so I was mixing myself up
anyways. in the present case, we've got $p(a)=A^{-1/2}\varphi(a/\sqrt{A})$ and $p(b|a)=B^{-1/2}\varphi((b-a)/\sqrt{B})$ where $\varphi(z)$ is the standard normal pdf
 
Someone can check the problem that i have trouble?
0
Q: Help for this problem involving rieman integral and partitions

Daniel MLIf $f: I--->\mathbb R$ is bounded, let $||f||:= Sup {|f(x)|: x \in I}$, and if $P =(x_0,...,x_n)$ is a partition of $I:=[a,b]$, let $||P||:=Sup [x_1-x_0,...,x_n-x_{n-1}]$ (a) If $P'$ is the partition obtained from $P$ as in the proof of Lemma 7.1.2, show that $L(P,f) ≤ L(P',f) ≤ L(P,f) + 2||f||...

 
so we're interested in $$p(b)=\int_{-\infty}^\infty \frac{1}{\sqrt{A}}\varphi\left(\frac{a}{\sqrt{A}}\right)\frac{1}{\sqrt{B}} \varphi \left(\frac{b-a}{\sqrt{B}}\right)\,da$$
 
Makes sense
 
actually, hmm
$p(b=x)=\sum_y p(b=x|a=y)p(a=y)$ makes sense for discrete r.v.s
But is that the right expression for continuous r.v's? you don't literally have $p(b=x|a=y)$ there
you have $p(b\leq x|a\leq y)$, for instance
but that's the cdf, not the pdf
 
Seems right actually
 

« first day (3031 days earlier)      last day (1991 days later) »